How to develop a solution in limit without L'Hopital?












2















Find $displaystylelim_{xto1}left(dfrac{1}{1-x}-dfrac{1}{ln x}right).$




I don't know how to remove $infty-infty$ uncertainty in the question. Please explain it and how many different solution can we do in the question?










share|cite|improve this question
























  • @copper.hat I put a picture link into sentence.You can look at it
    – Atakan
    Dec 9 '18 at 6:12










  • Are you familiar with L'Hôpital's rule? (The rule can be repeated as necessary.)
    – copper.hat
    Dec 9 '18 at 6:25








  • 1




    @Atakan: Limit does not exist check by $xto 1^-$ and $xto1^+$.
    – Yadati Kiran
    Dec 9 '18 at 6:28








  • 1




    @AniruddhVenkatesan: Yeah. as $xto1^-$ we have $+infty$ and $xto1^+$ we have $-infty$
    – Yadati Kiran
    Dec 9 '18 at 6:31








  • 1




    @Atakan: The two sided limit does not exist.
    – Yadati Kiran
    Dec 9 '18 at 6:34
















2















Find $displaystylelim_{xto1}left(dfrac{1}{1-x}-dfrac{1}{ln x}right).$




I don't know how to remove $infty-infty$ uncertainty in the question. Please explain it and how many different solution can we do in the question?










share|cite|improve this question
























  • @copper.hat I put a picture link into sentence.You can look at it
    – Atakan
    Dec 9 '18 at 6:12










  • Are you familiar with L'Hôpital's rule? (The rule can be repeated as necessary.)
    – copper.hat
    Dec 9 '18 at 6:25








  • 1




    @Atakan: Limit does not exist check by $xto 1^-$ and $xto1^+$.
    – Yadati Kiran
    Dec 9 '18 at 6:28








  • 1




    @AniruddhVenkatesan: Yeah. as $xto1^-$ we have $+infty$ and $xto1^+$ we have $-infty$
    – Yadati Kiran
    Dec 9 '18 at 6:31








  • 1




    @Atakan: The two sided limit does not exist.
    – Yadati Kiran
    Dec 9 '18 at 6:34














2












2








2








Find $displaystylelim_{xto1}left(dfrac{1}{1-x}-dfrac{1}{ln x}right).$




I don't know how to remove $infty-infty$ uncertainty in the question. Please explain it and how many different solution can we do in the question?










share|cite|improve this question
















Find $displaystylelim_{xto1}left(dfrac{1}{1-x}-dfrac{1}{ln x}right).$




I don't know how to remove $infty-infty$ uncertainty in the question. Please explain it and how many different solution can we do in the question?







calculus limits






share|cite|improve this question















share|cite|improve this question













share|cite|improve this question




share|cite|improve this question








edited Dec 9 '18 at 7:20

























asked Dec 9 '18 at 6:02









Atakan

184




184












  • @copper.hat I put a picture link into sentence.You can look at it
    – Atakan
    Dec 9 '18 at 6:12










  • Are you familiar with L'Hôpital's rule? (The rule can be repeated as necessary.)
    – copper.hat
    Dec 9 '18 at 6:25








  • 1




    @Atakan: Limit does not exist check by $xto 1^-$ and $xto1^+$.
    – Yadati Kiran
    Dec 9 '18 at 6:28








  • 1




    @AniruddhVenkatesan: Yeah. as $xto1^-$ we have $+infty$ and $xto1^+$ we have $-infty$
    – Yadati Kiran
    Dec 9 '18 at 6:31








  • 1




    @Atakan: The two sided limit does not exist.
    – Yadati Kiran
    Dec 9 '18 at 6:34


















  • @copper.hat I put a picture link into sentence.You can look at it
    – Atakan
    Dec 9 '18 at 6:12










  • Are you familiar with L'Hôpital's rule? (The rule can be repeated as necessary.)
    – copper.hat
    Dec 9 '18 at 6:25








  • 1




    @Atakan: Limit does not exist check by $xto 1^-$ and $xto1^+$.
    – Yadati Kiran
    Dec 9 '18 at 6:28








  • 1




    @AniruddhVenkatesan: Yeah. as $xto1^-$ we have $+infty$ and $xto1^+$ we have $-infty$
    – Yadati Kiran
    Dec 9 '18 at 6:31








  • 1




    @Atakan: The two sided limit does not exist.
    – Yadati Kiran
    Dec 9 '18 at 6:34
















@copper.hat I put a picture link into sentence.You can look at it
– Atakan
Dec 9 '18 at 6:12




@copper.hat I put a picture link into sentence.You can look at it
– Atakan
Dec 9 '18 at 6:12












Are you familiar with L'Hôpital's rule? (The rule can be repeated as necessary.)
– copper.hat
Dec 9 '18 at 6:25






Are you familiar with L'Hôpital's rule? (The rule can be repeated as necessary.)
– copper.hat
Dec 9 '18 at 6:25






1




1




@Atakan: Limit does not exist check by $xto 1^-$ and $xto1^+$.
– Yadati Kiran
Dec 9 '18 at 6:28






@Atakan: Limit does not exist check by $xto 1^-$ and $xto1^+$.
– Yadati Kiran
Dec 9 '18 at 6:28






1




1




@AniruddhVenkatesan: Yeah. as $xto1^-$ we have $+infty$ and $xto1^+$ we have $-infty$
– Yadati Kiran
Dec 9 '18 at 6:31






@AniruddhVenkatesan: Yeah. as $xto1^-$ we have $+infty$ and $xto1^+$ we have $-infty$
– Yadati Kiran
Dec 9 '18 at 6:31






1




1




@Atakan: The two sided limit does not exist.
– Yadati Kiran
Dec 9 '18 at 6:34




@Atakan: The two sided limit does not exist.
– Yadati Kiran
Dec 9 '18 at 6:34










5 Answers
5






active

oldest

votes


















4














You could try to use Taylor Series expansions.



$displaystylelim_{mto0}frac1m-frac1{ln(1-m)}, m=1-x$



$=displaystylelim_{mto0}frac1m+frac1{m+frac{m^2}2+frac{m^3}{3}...}=displaystylelim_{mto0}frac1m(1+frac1{1+frac m2+frac{m^2}3...})$



which diverges to $-infty$ when $mto0^-implies xto1^+$, and $+infty$ when $mto0^+implies xto1^-$.



enter image description here






share|cite|improve this answer























  • Ohh,I liked your solution and I'm supposing the book is wrong about shown answer.Because your solution is looking true
    – Atakan
    Dec 9 '18 at 6:43






  • 1




    The graph of $frac1{1-x}-frac1{ln x}$ supports my observation.
    – Shubham Johri
    Dec 9 '18 at 6:48










  • how did you prove it on the graph?Can you show it on the graph?
    – Atakan
    Dec 9 '18 at 6:51










  • thank you for showing it :)
    – Atakan
    Dec 9 '18 at 6:52






  • 1




    You're welcome.
    – Shubham Johri
    Dec 9 '18 at 6:54



















2














Write ${1 over 1-x} - {1 over log x} = {log x +x-1over (1-x) log x}$.



The Taylor expansions are
$log x +x-1 = 2(x-1)+cdots$ and $(1-x) log x = -(x-1)^2+ cdots $.



Hence we expect the behaviour for $x$ close to $1$ to be $approx -{2 over x-1}$.






share|cite|improve this answer





















  • Actually I couldn't understand what you mean,can you write as more detailed?
    – Atakan
    Dec 9 '18 at 7:06










  • Are you familiar with Taylor expansion?
    – copper.hat
    Dec 9 '18 at 7:10










  • no,I'm not familiar :(
    – Atakan
    Dec 9 '18 at 7:11










  • Certain functions can be expanded around a point in the form $f(x) = sum_{k ge 0} f^{(k)}(x_o) (x-x_0)^k$. (en.wikipedia.org/wiki/Taylor_series.) This expansion can be used to approximate the local behaviour of $f$. In this case, I am approximating the numerator and denominator by Taylor series, and the leading (non zero) terms dictate the quotient's behaviour.
    – copper.hat
    Dec 9 '18 at 7:15












  • okay, I'm trying to understand it
    – Atakan
    Dec 9 '18 at 7:24



















2














I want to rewrite the limit slightly:



$$lim_{xto1} left(frac{1}{1-x} - frac{1}{ln(x)}right) = -lim_{xto1} left(frac{1}{ln(x)} + frac{1}{x-1} right). $$



Note that now our two summands have the same sign when $x<1$ and when $x>1$. The limit from the left goes to $(-infty)+ (-infty) = (-infty)$ and the limit from the right goes to $infty + infty = infty$. Thus, the limit does not exist.



No indefinite forms needed!






share|cite|improve this answer





















  • Yes,you solved it with small changes but it's looking true ,thank you for your solving :)
    – Atakan
    Dec 9 '18 at 7:08










  • That's the best solution in my opinion! Simple and clever.
    – gimusi
    Dec 9 '18 at 14:49





















2














I guess the first term should have been $frac{1}{x-1}$ rather than $frac{1}{1-x}$. In the former case the substitution $x=e^t$ reduces the problem to the evaluation of
$$ lim_{tto 0}left(frac{1}{e^t-1}-frac{1}{t}right)=-lim_{t to 0}frac{e^t-1-t}{t(e^t-1)}=-lim_{tto 0}frac{frac{t^2}{2}+O(t^3)}{t^2+O(t^3)}=color{red}{-frac{1}{2}}. $$
The Maclaurin series of $e^t$ is either a trivial consequence of the series definition of $e^t$ or a simple consequence of the dominated convergence theorem / integration by parts:
$$ e^t-1-t = t^2int_{0}^{1}(1-x)e^{tx},dx to t^2 int_{0}^{1}(1-x),dx = frac{t^2}{2}.$$






share|cite|improve this answer





























    2














    We have that by $y=x-1 to 0$



    $$lim_{xto1} left(dfrac{1}{1-x}-dfrac{1}{ln x}right) =lim_{yto0} left(-dfrac{1}{y}-dfrac{1}{ln (1+y)}right)$$



    and by standard limits since $frac{ln(1+y)}yto 1$



    $$-dfrac{1}{y}-dfrac{1}{ln (1+y)}=-dfrac{ln(1+y)+y}{yln (1+y)}=-dfrac{frac{ln(1+y)}y+1}{ln (1+y)}to begin{cases}-inftyquad yto 0^+\\+inftyquad yto 0^- end{cases}$$






    share|cite|improve this answer























    • $+infty$ when $yto0^-$
      – Shubham Johri
      Dec 9 '18 at 12:28












    • @ShubhamJohri Yes indeed I lost a minus sign in the last step and reverse the result! Thanks so much for pointing out that!
      – gimusi
      Dec 9 '18 at 12:33












    • You're welcome ;)
      – Shubham Johri
      Dec 9 '18 at 12:35











    Your Answer





    StackExchange.ifUsing("editor", function () {
    return StackExchange.using("mathjaxEditing", function () {
    StackExchange.MarkdownEditor.creationCallbacks.add(function (editor, postfix) {
    StackExchange.mathjaxEditing.prepareWmdForMathJax(editor, postfix, [["$", "$"], ["\\(","\\)"]]);
    });
    });
    }, "mathjax-editing");

    StackExchange.ready(function() {
    var channelOptions = {
    tags: "".split(" "),
    id: "69"
    };
    initTagRenderer("".split(" "), "".split(" "), channelOptions);

    StackExchange.using("externalEditor", function() {
    // Have to fire editor after snippets, if snippets enabled
    if (StackExchange.settings.snippets.snippetsEnabled) {
    StackExchange.using("snippets", function() {
    createEditor();
    });
    }
    else {
    createEditor();
    }
    });

    function createEditor() {
    StackExchange.prepareEditor({
    heartbeatType: 'answer',
    autoActivateHeartbeat: false,
    convertImagesToLinks: true,
    noModals: true,
    showLowRepImageUploadWarning: true,
    reputationToPostImages: 10,
    bindNavPrevention: true,
    postfix: "",
    imageUploader: {
    brandingHtml: "Powered by u003ca class="icon-imgur-white" href="https://imgur.com/"u003eu003c/au003e",
    contentPolicyHtml: "User contributions licensed under u003ca href="https://creativecommons.org/licenses/by-sa/3.0/"u003ecc by-sa 3.0 with attribution requiredu003c/au003e u003ca href="https://stackoverflow.com/legal/content-policy"u003e(content policy)u003c/au003e",
    allowUrls: true
    },
    noCode: true, onDemand: true,
    discardSelector: ".discard-answer"
    ,immediatelyShowMarkdownHelp:true
    });


    }
    });














    draft saved

    draft discarded


















    StackExchange.ready(
    function () {
    StackExchange.openid.initPostLogin('.new-post-login', 'https%3a%2f%2fmath.stackexchange.com%2fquestions%2f3032067%2fhow-to-develop-a-solution-in-limit-without-lhopital%23new-answer', 'question_page');
    }
    );

    Post as a guest















    Required, but never shown

























    5 Answers
    5






    active

    oldest

    votes








    5 Answers
    5






    active

    oldest

    votes









    active

    oldest

    votes






    active

    oldest

    votes









    4














    You could try to use Taylor Series expansions.



    $displaystylelim_{mto0}frac1m-frac1{ln(1-m)}, m=1-x$



    $=displaystylelim_{mto0}frac1m+frac1{m+frac{m^2}2+frac{m^3}{3}...}=displaystylelim_{mto0}frac1m(1+frac1{1+frac m2+frac{m^2}3...})$



    which diverges to $-infty$ when $mto0^-implies xto1^+$, and $+infty$ when $mto0^+implies xto1^-$.



    enter image description here






    share|cite|improve this answer























    • Ohh,I liked your solution and I'm supposing the book is wrong about shown answer.Because your solution is looking true
      – Atakan
      Dec 9 '18 at 6:43






    • 1




      The graph of $frac1{1-x}-frac1{ln x}$ supports my observation.
      – Shubham Johri
      Dec 9 '18 at 6:48










    • how did you prove it on the graph?Can you show it on the graph?
      – Atakan
      Dec 9 '18 at 6:51










    • thank you for showing it :)
      – Atakan
      Dec 9 '18 at 6:52






    • 1




      You're welcome.
      – Shubham Johri
      Dec 9 '18 at 6:54
















    4














    You could try to use Taylor Series expansions.



    $displaystylelim_{mto0}frac1m-frac1{ln(1-m)}, m=1-x$



    $=displaystylelim_{mto0}frac1m+frac1{m+frac{m^2}2+frac{m^3}{3}...}=displaystylelim_{mto0}frac1m(1+frac1{1+frac m2+frac{m^2}3...})$



    which diverges to $-infty$ when $mto0^-implies xto1^+$, and $+infty$ when $mto0^+implies xto1^-$.



    enter image description here






    share|cite|improve this answer























    • Ohh,I liked your solution and I'm supposing the book is wrong about shown answer.Because your solution is looking true
      – Atakan
      Dec 9 '18 at 6:43






    • 1




      The graph of $frac1{1-x}-frac1{ln x}$ supports my observation.
      – Shubham Johri
      Dec 9 '18 at 6:48










    • how did you prove it on the graph?Can you show it on the graph?
      – Atakan
      Dec 9 '18 at 6:51










    • thank you for showing it :)
      – Atakan
      Dec 9 '18 at 6:52






    • 1




      You're welcome.
      – Shubham Johri
      Dec 9 '18 at 6:54














    4












    4








    4






    You could try to use Taylor Series expansions.



    $displaystylelim_{mto0}frac1m-frac1{ln(1-m)}, m=1-x$



    $=displaystylelim_{mto0}frac1m+frac1{m+frac{m^2}2+frac{m^3}{3}...}=displaystylelim_{mto0}frac1m(1+frac1{1+frac m2+frac{m^2}3...})$



    which diverges to $-infty$ when $mto0^-implies xto1^+$, and $+infty$ when $mto0^+implies xto1^-$.



    enter image description here






    share|cite|improve this answer














    You could try to use Taylor Series expansions.



    $displaystylelim_{mto0}frac1m-frac1{ln(1-m)}, m=1-x$



    $=displaystylelim_{mto0}frac1m+frac1{m+frac{m^2}2+frac{m^3}{3}...}=displaystylelim_{mto0}frac1m(1+frac1{1+frac m2+frac{m^2}3...})$



    which diverges to $-infty$ when $mto0^-implies xto1^+$, and $+infty$ when $mto0^+implies xto1^-$.



    enter image description here







    share|cite|improve this answer














    share|cite|improve this answer



    share|cite|improve this answer








    edited Dec 9 '18 at 12:26

























    answered Dec 9 '18 at 6:37









    Shubham Johri

    3,961717




    3,961717












    • Ohh,I liked your solution and I'm supposing the book is wrong about shown answer.Because your solution is looking true
      – Atakan
      Dec 9 '18 at 6:43






    • 1




      The graph of $frac1{1-x}-frac1{ln x}$ supports my observation.
      – Shubham Johri
      Dec 9 '18 at 6:48










    • how did you prove it on the graph?Can you show it on the graph?
      – Atakan
      Dec 9 '18 at 6:51










    • thank you for showing it :)
      – Atakan
      Dec 9 '18 at 6:52






    • 1




      You're welcome.
      – Shubham Johri
      Dec 9 '18 at 6:54


















    • Ohh,I liked your solution and I'm supposing the book is wrong about shown answer.Because your solution is looking true
      – Atakan
      Dec 9 '18 at 6:43






    • 1




      The graph of $frac1{1-x}-frac1{ln x}$ supports my observation.
      – Shubham Johri
      Dec 9 '18 at 6:48










    • how did you prove it on the graph?Can you show it on the graph?
      – Atakan
      Dec 9 '18 at 6:51










    • thank you for showing it :)
      – Atakan
      Dec 9 '18 at 6:52






    • 1




      You're welcome.
      – Shubham Johri
      Dec 9 '18 at 6:54
















    Ohh,I liked your solution and I'm supposing the book is wrong about shown answer.Because your solution is looking true
    – Atakan
    Dec 9 '18 at 6:43




    Ohh,I liked your solution and I'm supposing the book is wrong about shown answer.Because your solution is looking true
    – Atakan
    Dec 9 '18 at 6:43




    1




    1




    The graph of $frac1{1-x}-frac1{ln x}$ supports my observation.
    – Shubham Johri
    Dec 9 '18 at 6:48




    The graph of $frac1{1-x}-frac1{ln x}$ supports my observation.
    – Shubham Johri
    Dec 9 '18 at 6:48












    how did you prove it on the graph?Can you show it on the graph?
    – Atakan
    Dec 9 '18 at 6:51




    how did you prove it on the graph?Can you show it on the graph?
    – Atakan
    Dec 9 '18 at 6:51












    thank you for showing it :)
    – Atakan
    Dec 9 '18 at 6:52




    thank you for showing it :)
    – Atakan
    Dec 9 '18 at 6:52




    1




    1




    You're welcome.
    – Shubham Johri
    Dec 9 '18 at 6:54




    You're welcome.
    – Shubham Johri
    Dec 9 '18 at 6:54











    2














    Write ${1 over 1-x} - {1 over log x} = {log x +x-1over (1-x) log x}$.



    The Taylor expansions are
    $log x +x-1 = 2(x-1)+cdots$ and $(1-x) log x = -(x-1)^2+ cdots $.



    Hence we expect the behaviour for $x$ close to $1$ to be $approx -{2 over x-1}$.






    share|cite|improve this answer





















    • Actually I couldn't understand what you mean,can you write as more detailed?
      – Atakan
      Dec 9 '18 at 7:06










    • Are you familiar with Taylor expansion?
      – copper.hat
      Dec 9 '18 at 7:10










    • no,I'm not familiar :(
      – Atakan
      Dec 9 '18 at 7:11










    • Certain functions can be expanded around a point in the form $f(x) = sum_{k ge 0} f^{(k)}(x_o) (x-x_0)^k$. (en.wikipedia.org/wiki/Taylor_series.) This expansion can be used to approximate the local behaviour of $f$. In this case, I am approximating the numerator and denominator by Taylor series, and the leading (non zero) terms dictate the quotient's behaviour.
      – copper.hat
      Dec 9 '18 at 7:15












    • okay, I'm trying to understand it
      – Atakan
      Dec 9 '18 at 7:24
















    2














    Write ${1 over 1-x} - {1 over log x} = {log x +x-1over (1-x) log x}$.



    The Taylor expansions are
    $log x +x-1 = 2(x-1)+cdots$ and $(1-x) log x = -(x-1)^2+ cdots $.



    Hence we expect the behaviour for $x$ close to $1$ to be $approx -{2 over x-1}$.






    share|cite|improve this answer





















    • Actually I couldn't understand what you mean,can you write as more detailed?
      – Atakan
      Dec 9 '18 at 7:06










    • Are you familiar with Taylor expansion?
      – copper.hat
      Dec 9 '18 at 7:10










    • no,I'm not familiar :(
      – Atakan
      Dec 9 '18 at 7:11










    • Certain functions can be expanded around a point in the form $f(x) = sum_{k ge 0} f^{(k)}(x_o) (x-x_0)^k$. (en.wikipedia.org/wiki/Taylor_series.) This expansion can be used to approximate the local behaviour of $f$. In this case, I am approximating the numerator and denominator by Taylor series, and the leading (non zero) terms dictate the quotient's behaviour.
      – copper.hat
      Dec 9 '18 at 7:15












    • okay, I'm trying to understand it
      – Atakan
      Dec 9 '18 at 7:24














    2












    2








    2






    Write ${1 over 1-x} - {1 over log x} = {log x +x-1over (1-x) log x}$.



    The Taylor expansions are
    $log x +x-1 = 2(x-1)+cdots$ and $(1-x) log x = -(x-1)^2+ cdots $.



    Hence we expect the behaviour for $x$ close to $1$ to be $approx -{2 over x-1}$.






    share|cite|improve this answer












    Write ${1 over 1-x} - {1 over log x} = {log x +x-1over (1-x) log x}$.



    The Taylor expansions are
    $log x +x-1 = 2(x-1)+cdots$ and $(1-x) log x = -(x-1)^2+ cdots $.



    Hence we expect the behaviour for $x$ close to $1$ to be $approx -{2 over x-1}$.







    share|cite|improve this answer












    share|cite|improve this answer



    share|cite|improve this answer










    answered Dec 9 '18 at 6:56









    copper.hat

    126k559159




    126k559159












    • Actually I couldn't understand what you mean,can you write as more detailed?
      – Atakan
      Dec 9 '18 at 7:06










    • Are you familiar with Taylor expansion?
      – copper.hat
      Dec 9 '18 at 7:10










    • no,I'm not familiar :(
      – Atakan
      Dec 9 '18 at 7:11










    • Certain functions can be expanded around a point in the form $f(x) = sum_{k ge 0} f^{(k)}(x_o) (x-x_0)^k$. (en.wikipedia.org/wiki/Taylor_series.) This expansion can be used to approximate the local behaviour of $f$. In this case, I am approximating the numerator and denominator by Taylor series, and the leading (non zero) terms dictate the quotient's behaviour.
      – copper.hat
      Dec 9 '18 at 7:15












    • okay, I'm trying to understand it
      – Atakan
      Dec 9 '18 at 7:24


















    • Actually I couldn't understand what you mean,can you write as more detailed?
      – Atakan
      Dec 9 '18 at 7:06










    • Are you familiar with Taylor expansion?
      – copper.hat
      Dec 9 '18 at 7:10










    • no,I'm not familiar :(
      – Atakan
      Dec 9 '18 at 7:11










    • Certain functions can be expanded around a point in the form $f(x) = sum_{k ge 0} f^{(k)}(x_o) (x-x_0)^k$. (en.wikipedia.org/wiki/Taylor_series.) This expansion can be used to approximate the local behaviour of $f$. In this case, I am approximating the numerator and denominator by Taylor series, and the leading (non zero) terms dictate the quotient's behaviour.
      – copper.hat
      Dec 9 '18 at 7:15












    • okay, I'm trying to understand it
      – Atakan
      Dec 9 '18 at 7:24
















    Actually I couldn't understand what you mean,can you write as more detailed?
    – Atakan
    Dec 9 '18 at 7:06




    Actually I couldn't understand what you mean,can you write as more detailed?
    – Atakan
    Dec 9 '18 at 7:06












    Are you familiar with Taylor expansion?
    – copper.hat
    Dec 9 '18 at 7:10




    Are you familiar with Taylor expansion?
    – copper.hat
    Dec 9 '18 at 7:10












    no,I'm not familiar :(
    – Atakan
    Dec 9 '18 at 7:11




    no,I'm not familiar :(
    – Atakan
    Dec 9 '18 at 7:11












    Certain functions can be expanded around a point in the form $f(x) = sum_{k ge 0} f^{(k)}(x_o) (x-x_0)^k$. (en.wikipedia.org/wiki/Taylor_series.) This expansion can be used to approximate the local behaviour of $f$. In this case, I am approximating the numerator and denominator by Taylor series, and the leading (non zero) terms dictate the quotient's behaviour.
    – copper.hat
    Dec 9 '18 at 7:15






    Certain functions can be expanded around a point in the form $f(x) = sum_{k ge 0} f^{(k)}(x_o) (x-x_0)^k$. (en.wikipedia.org/wiki/Taylor_series.) This expansion can be used to approximate the local behaviour of $f$. In this case, I am approximating the numerator and denominator by Taylor series, and the leading (non zero) terms dictate the quotient's behaviour.
    – copper.hat
    Dec 9 '18 at 7:15














    okay, I'm trying to understand it
    – Atakan
    Dec 9 '18 at 7:24




    okay, I'm trying to understand it
    – Atakan
    Dec 9 '18 at 7:24











    2














    I want to rewrite the limit slightly:



    $$lim_{xto1} left(frac{1}{1-x} - frac{1}{ln(x)}right) = -lim_{xto1} left(frac{1}{ln(x)} + frac{1}{x-1} right). $$



    Note that now our two summands have the same sign when $x<1$ and when $x>1$. The limit from the left goes to $(-infty)+ (-infty) = (-infty)$ and the limit from the right goes to $infty + infty = infty$. Thus, the limit does not exist.



    No indefinite forms needed!






    share|cite|improve this answer





















    • Yes,you solved it with small changes but it's looking true ,thank you for your solving :)
      – Atakan
      Dec 9 '18 at 7:08










    • That's the best solution in my opinion! Simple and clever.
      – gimusi
      Dec 9 '18 at 14:49


















    2














    I want to rewrite the limit slightly:



    $$lim_{xto1} left(frac{1}{1-x} - frac{1}{ln(x)}right) = -lim_{xto1} left(frac{1}{ln(x)} + frac{1}{x-1} right). $$



    Note that now our two summands have the same sign when $x<1$ and when $x>1$. The limit from the left goes to $(-infty)+ (-infty) = (-infty)$ and the limit from the right goes to $infty + infty = infty$. Thus, the limit does not exist.



    No indefinite forms needed!






    share|cite|improve this answer





















    • Yes,you solved it with small changes but it's looking true ,thank you for your solving :)
      – Atakan
      Dec 9 '18 at 7:08










    • That's the best solution in my opinion! Simple and clever.
      – gimusi
      Dec 9 '18 at 14:49
















    2












    2








    2






    I want to rewrite the limit slightly:



    $$lim_{xto1} left(frac{1}{1-x} - frac{1}{ln(x)}right) = -lim_{xto1} left(frac{1}{ln(x)} + frac{1}{x-1} right). $$



    Note that now our two summands have the same sign when $x<1$ and when $x>1$. The limit from the left goes to $(-infty)+ (-infty) = (-infty)$ and the limit from the right goes to $infty + infty = infty$. Thus, the limit does not exist.



    No indefinite forms needed!






    share|cite|improve this answer












    I want to rewrite the limit slightly:



    $$lim_{xto1} left(frac{1}{1-x} - frac{1}{ln(x)}right) = -lim_{xto1} left(frac{1}{ln(x)} + frac{1}{x-1} right). $$



    Note that now our two summands have the same sign when $x<1$ and when $x>1$. The limit from the left goes to $(-infty)+ (-infty) = (-infty)$ and the limit from the right goes to $infty + infty = infty$. Thus, the limit does not exist.



    No indefinite forms needed!







    share|cite|improve this answer












    share|cite|improve this answer



    share|cite|improve this answer










    answered Dec 9 '18 at 6:57









    Santana Afton

    2,5742629




    2,5742629












    • Yes,you solved it with small changes but it's looking true ,thank you for your solving :)
      – Atakan
      Dec 9 '18 at 7:08










    • That's the best solution in my opinion! Simple and clever.
      – gimusi
      Dec 9 '18 at 14:49




















    • Yes,you solved it with small changes but it's looking true ,thank you for your solving :)
      – Atakan
      Dec 9 '18 at 7:08










    • That's the best solution in my opinion! Simple and clever.
      – gimusi
      Dec 9 '18 at 14:49


















    Yes,you solved it with small changes but it's looking true ,thank you for your solving :)
    – Atakan
    Dec 9 '18 at 7:08




    Yes,you solved it with small changes but it's looking true ,thank you for your solving :)
    – Atakan
    Dec 9 '18 at 7:08












    That's the best solution in my opinion! Simple and clever.
    – gimusi
    Dec 9 '18 at 14:49






    That's the best solution in my opinion! Simple and clever.
    – gimusi
    Dec 9 '18 at 14:49













    2














    I guess the first term should have been $frac{1}{x-1}$ rather than $frac{1}{1-x}$. In the former case the substitution $x=e^t$ reduces the problem to the evaluation of
    $$ lim_{tto 0}left(frac{1}{e^t-1}-frac{1}{t}right)=-lim_{t to 0}frac{e^t-1-t}{t(e^t-1)}=-lim_{tto 0}frac{frac{t^2}{2}+O(t^3)}{t^2+O(t^3)}=color{red}{-frac{1}{2}}. $$
    The Maclaurin series of $e^t$ is either a trivial consequence of the series definition of $e^t$ or a simple consequence of the dominated convergence theorem / integration by parts:
    $$ e^t-1-t = t^2int_{0}^{1}(1-x)e^{tx},dx to t^2 int_{0}^{1}(1-x),dx = frac{t^2}{2}.$$






    share|cite|improve this answer


























      2














      I guess the first term should have been $frac{1}{x-1}$ rather than $frac{1}{1-x}$. In the former case the substitution $x=e^t$ reduces the problem to the evaluation of
      $$ lim_{tto 0}left(frac{1}{e^t-1}-frac{1}{t}right)=-lim_{t to 0}frac{e^t-1-t}{t(e^t-1)}=-lim_{tto 0}frac{frac{t^2}{2}+O(t^3)}{t^2+O(t^3)}=color{red}{-frac{1}{2}}. $$
      The Maclaurin series of $e^t$ is either a trivial consequence of the series definition of $e^t$ or a simple consequence of the dominated convergence theorem / integration by parts:
      $$ e^t-1-t = t^2int_{0}^{1}(1-x)e^{tx},dx to t^2 int_{0}^{1}(1-x),dx = frac{t^2}{2}.$$






      share|cite|improve this answer
























        2












        2








        2






        I guess the first term should have been $frac{1}{x-1}$ rather than $frac{1}{1-x}$. In the former case the substitution $x=e^t$ reduces the problem to the evaluation of
        $$ lim_{tto 0}left(frac{1}{e^t-1}-frac{1}{t}right)=-lim_{t to 0}frac{e^t-1-t}{t(e^t-1)}=-lim_{tto 0}frac{frac{t^2}{2}+O(t^3)}{t^2+O(t^3)}=color{red}{-frac{1}{2}}. $$
        The Maclaurin series of $e^t$ is either a trivial consequence of the series definition of $e^t$ or a simple consequence of the dominated convergence theorem / integration by parts:
        $$ e^t-1-t = t^2int_{0}^{1}(1-x)e^{tx},dx to t^2 int_{0}^{1}(1-x),dx = frac{t^2}{2}.$$






        share|cite|improve this answer












        I guess the first term should have been $frac{1}{x-1}$ rather than $frac{1}{1-x}$. In the former case the substitution $x=e^t$ reduces the problem to the evaluation of
        $$ lim_{tto 0}left(frac{1}{e^t-1}-frac{1}{t}right)=-lim_{t to 0}frac{e^t-1-t}{t(e^t-1)}=-lim_{tto 0}frac{frac{t^2}{2}+O(t^3)}{t^2+O(t^3)}=color{red}{-frac{1}{2}}. $$
        The Maclaurin series of $e^t$ is either a trivial consequence of the series definition of $e^t$ or a simple consequence of the dominated convergence theorem / integration by parts:
        $$ e^t-1-t = t^2int_{0}^{1}(1-x)e^{tx},dx to t^2 int_{0}^{1}(1-x),dx = frac{t^2}{2}.$$







        share|cite|improve this answer












        share|cite|improve this answer



        share|cite|improve this answer










        answered Dec 9 '18 at 11:45









        Jack D'Aurizio

        287k33280657




        287k33280657























            2














            We have that by $y=x-1 to 0$



            $$lim_{xto1} left(dfrac{1}{1-x}-dfrac{1}{ln x}right) =lim_{yto0} left(-dfrac{1}{y}-dfrac{1}{ln (1+y)}right)$$



            and by standard limits since $frac{ln(1+y)}yto 1$



            $$-dfrac{1}{y}-dfrac{1}{ln (1+y)}=-dfrac{ln(1+y)+y}{yln (1+y)}=-dfrac{frac{ln(1+y)}y+1}{ln (1+y)}to begin{cases}-inftyquad yto 0^+\\+inftyquad yto 0^- end{cases}$$






            share|cite|improve this answer























            • $+infty$ when $yto0^-$
              – Shubham Johri
              Dec 9 '18 at 12:28












            • @ShubhamJohri Yes indeed I lost a minus sign in the last step and reverse the result! Thanks so much for pointing out that!
              – gimusi
              Dec 9 '18 at 12:33












            • You're welcome ;)
              – Shubham Johri
              Dec 9 '18 at 12:35
















            2














            We have that by $y=x-1 to 0$



            $$lim_{xto1} left(dfrac{1}{1-x}-dfrac{1}{ln x}right) =lim_{yto0} left(-dfrac{1}{y}-dfrac{1}{ln (1+y)}right)$$



            and by standard limits since $frac{ln(1+y)}yto 1$



            $$-dfrac{1}{y}-dfrac{1}{ln (1+y)}=-dfrac{ln(1+y)+y}{yln (1+y)}=-dfrac{frac{ln(1+y)}y+1}{ln (1+y)}to begin{cases}-inftyquad yto 0^+\\+inftyquad yto 0^- end{cases}$$






            share|cite|improve this answer























            • $+infty$ when $yto0^-$
              – Shubham Johri
              Dec 9 '18 at 12:28












            • @ShubhamJohri Yes indeed I lost a minus sign in the last step and reverse the result! Thanks so much for pointing out that!
              – gimusi
              Dec 9 '18 at 12:33












            • You're welcome ;)
              – Shubham Johri
              Dec 9 '18 at 12:35














            2












            2








            2






            We have that by $y=x-1 to 0$



            $$lim_{xto1} left(dfrac{1}{1-x}-dfrac{1}{ln x}right) =lim_{yto0} left(-dfrac{1}{y}-dfrac{1}{ln (1+y)}right)$$



            and by standard limits since $frac{ln(1+y)}yto 1$



            $$-dfrac{1}{y}-dfrac{1}{ln (1+y)}=-dfrac{ln(1+y)+y}{yln (1+y)}=-dfrac{frac{ln(1+y)}y+1}{ln (1+y)}to begin{cases}-inftyquad yto 0^+\\+inftyquad yto 0^- end{cases}$$






            share|cite|improve this answer














            We have that by $y=x-1 to 0$



            $$lim_{xto1} left(dfrac{1}{1-x}-dfrac{1}{ln x}right) =lim_{yto0} left(-dfrac{1}{y}-dfrac{1}{ln (1+y)}right)$$



            and by standard limits since $frac{ln(1+y)}yto 1$



            $$-dfrac{1}{y}-dfrac{1}{ln (1+y)}=-dfrac{ln(1+y)+y}{yln (1+y)}=-dfrac{frac{ln(1+y)}y+1}{ln (1+y)}to begin{cases}-inftyquad yto 0^+\\+inftyquad yto 0^- end{cases}$$







            share|cite|improve this answer














            share|cite|improve this answer



            share|cite|improve this answer








            edited Dec 9 '18 at 12:34

























            answered Dec 9 '18 at 8:12









            gimusi

            1




            1












            • $+infty$ when $yto0^-$
              – Shubham Johri
              Dec 9 '18 at 12:28












            • @ShubhamJohri Yes indeed I lost a minus sign in the last step and reverse the result! Thanks so much for pointing out that!
              – gimusi
              Dec 9 '18 at 12:33












            • You're welcome ;)
              – Shubham Johri
              Dec 9 '18 at 12:35


















            • $+infty$ when $yto0^-$
              – Shubham Johri
              Dec 9 '18 at 12:28












            • @ShubhamJohri Yes indeed I lost a minus sign in the last step and reverse the result! Thanks so much for pointing out that!
              – gimusi
              Dec 9 '18 at 12:33












            • You're welcome ;)
              – Shubham Johri
              Dec 9 '18 at 12:35
















            $+infty$ when $yto0^-$
            – Shubham Johri
            Dec 9 '18 at 12:28






            $+infty$ when $yto0^-$
            – Shubham Johri
            Dec 9 '18 at 12:28














            @ShubhamJohri Yes indeed I lost a minus sign in the last step and reverse the result! Thanks so much for pointing out that!
            – gimusi
            Dec 9 '18 at 12:33






            @ShubhamJohri Yes indeed I lost a minus sign in the last step and reverse the result! Thanks so much for pointing out that!
            – gimusi
            Dec 9 '18 at 12:33














            You're welcome ;)
            – Shubham Johri
            Dec 9 '18 at 12:35




            You're welcome ;)
            – Shubham Johri
            Dec 9 '18 at 12:35


















            draft saved

            draft discarded




















































            Thanks for contributing an answer to Mathematics Stack Exchange!


            • Please be sure to answer the question. Provide details and share your research!

            But avoid



            • Asking for help, clarification, or responding to other answers.

            • Making statements based on opinion; back them up with references or personal experience.


            Use MathJax to format equations. MathJax reference.


            To learn more, see our tips on writing great answers.





            Some of your past answers have not been well-received, and you're in danger of being blocked from answering.


            Please pay close attention to the following guidance:


            • Please be sure to answer the question. Provide details and share your research!

            But avoid



            • Asking for help, clarification, or responding to other answers.

            • Making statements based on opinion; back them up with references or personal experience.


            To learn more, see our tips on writing great answers.




            draft saved


            draft discarded














            StackExchange.ready(
            function () {
            StackExchange.openid.initPostLogin('.new-post-login', 'https%3a%2f%2fmath.stackexchange.com%2fquestions%2f3032067%2fhow-to-develop-a-solution-in-limit-without-lhopital%23new-answer', 'question_page');
            }
            );

            Post as a guest















            Required, but never shown





















































            Required, but never shown














            Required, but never shown












            Required, but never shown







            Required, but never shown

































            Required, but never shown














            Required, but never shown












            Required, but never shown







            Required, but never shown







            Popular posts from this blog

            Список кардиналов, возведённых папой римским Каликстом III

            Deduzione

            Mysql.sock missing - “Can't connect to local MySQL server through socket”